Download as pdf or txt
Download as pdf or txt
You are on page 1of 24

CIVIL SERVICES (PRELIMINARY) TEST SERIES – 2019

TEST BOOKLET SERIES


ANSWER & EXPLANATION
Test Code:
P1-1923 GENERAL STUDIES
(English Version) TEST – 23

ANSWERS
Q. No. Answers Q. No. Answers Q. No. Answers Q. No. Answers
1 (a) 26 (d) 51 (a) 76 (a)
2 (b) 27 (b) 52 (d) 77 (b)
3 (d) 28 (d) 53 (c) 78 (c)
4 (d) 29 (a) 54 (c) 79 (c)
5 (b) 30 (c) 55 (c) 80 (b)
6 (d) 31 (b) 56 (c) 81 (c)
7 (c) 32 (c) 57 (c) 82 (b)
8 (b) 33 (b) 58 (c) 83 (d)
9 (c) 34 (a) 59 (c) 84 (a)
10 (d) 35 (a) 60 (c) 85 (c)
11 (d) 36 (a) 61 (a) 86 (c)
12 (a) 37 (b) 62 (d) 87 (d)

13 (b) 38 (d) 63 (a) 88 (c)

14 (c) 39 (c) 64 (d) 89 (d)


15 (a) 40 (a) 65 (b) 90 (d)
16 (a) 41 (a) 66 (b) 91 (c)
17 (b) 42 (b) 67 (c) 92 (a)
18 (d) 43 (a) 68 (a) 93 (d)
19 (c) 44 (c) 69 (b) 94 (d)
20 (d) 45 (d) 70 (c) 95 (c)
21 (a) 46 (c) 71 (d) 96 (a)
22 (a) 47 (d) 72 (b) 97 (d)
23 (c) 48 (b) 73 (a) 98 (a)
24 (d) 49 (d) 74 (b) 99 (c)
25 (a) 50 (d) 75 (d) 100 (a)
1. (a) 4. (d)
Exp.: India is now a member of the Missile Technology Control Exp.: The Tata Trusts and the World Council on City Data
Regime (MTCR), the Wassenaar Arrangement (WA) as (WCCD) have established a major partnership to deliver
well as the Australia Group AG, three of four non- the City Data for India Initiative. Hence, option (d) is
proliferation regimes. correct.
zzThe only one remaining is the Nuclear Suppliers Group zzThe goal of the City Data for India Initiative is to
(NSG). Hence, option (a) is correct. contribute to improved infrastructure services, inclusive
prosperity and quality of life for millions of Indian urban
2. (b)
citizens.
Exp.: Consumption of Used Cooking Oil (UCO) poses adverse
zzThe Initiative empowers city leaders, decision-makers
health effects. During frying, several properties of oil are
and citizens from major cities in India to make data-
altered, Total Polar Compounds (TPC) are formed on
informed decisions to improve city planning, infrastructure
repeated frying.
investment and operational performance management
zzThe Food Safety and Standards Authority of India
using internationally standardized and comparable open
(FSSAI) has launched RUCO (Repurpose Used Cooking
city data.
Oil), an initiative that will enable collection and
conversion of used cooking oil to biodiesel. Hence, zzThis initiative will complement two flagship urban
statement 1 is NOT correct. development programmes, the ‘AMRUT’ and ‘Smart
zzRepurpose Used Cooking Oil (RUCO) is an ecosystem
Cities’ mission.
that will enable the collection and conversion of Used 5. (b)
Cooking Oil (UCO) to biodiesel. Hence, statement 2 is
Exp.: The Aurora is a light show caused by collisions
correct.
between electrically charged particles released from
zzThe toxicity of these compounds is associated with several
the sun that enter the earth’s atmosphere and collide
diseases such as hypertension, atherosclerosis, Alzheimer’s
with gases such as oxygen, nitrogen. Hence, statement
disease, liver diseases. Therefore, it is essential to monitor
2 is correct.
the quality of vegetable oils during frying.
zzThe lights are seen around the magnetic poles of the
zzIn order to safeguard consumer health, FSSAI has fixed northern and southern hemispheres.
a limit for Total Polar Compounds at 25 percent beyond
zzAuroras that occur in the northern hemisphere are called
which the vegetable oil shall not be used. From 1st July,
‘Aurora Borealis’ or ‘northern lights’ and auroras that
2018 onwards, all Food Business Operators (FBOs) are
occur in the southern hemisphere are called ‘Aurora
required to monitor the quality of oil during frying by
Australis’ or ‘southern lights’. Hence, statement 1 is
complying with the said regulations.
NOT correct.
zzUnder this initiative, 64 companies at 101 locations have
zzThe auroras can appear in many forms, from small patches
been identified to enable collection of used cooking oil.
of light to streamers, arcs, rippling curtains or shooting
For instance: McDonald’s has already started converting
rays that light up the sky with glow.
used cooking oil to biodiesel from 100 outlets in Mumbai
and Pune. zzThey commonly occur at high northern and southern
latitudes, less frequent at mid-latitudes, and seldom seen
3. (d) near the equator.
Exp.: The Economist Intelligence Unit's liveability rating 6. (d)
(Global Liveability Index) quantifies the challenges that
might be presented to an individual's lifestyle in 140 cities Exp.: Greenhouse gases (GHGs) warm the Earth by
worldwide. Hence, option (d) is correct. absorbing energy and slowing the rate at which the
zzEach city is assigned a score for over 30 qualitative and energy escapes to space; they act like a blanket
quantitative factors across five broad categories of insulating the Earth.
Stability, Healthcare, Culture and environment, Education zzThe Global Warming Potential (GWP) allows comparisons
and Infrastructure. of the global warming impacts of different gases.
zzDelhi and Mumbai are only two Indian cities that are zzIt is a measure of how much energy the emissions of 1
included in EIU’s Indore with rank of 112 and 117 ton of a gas will absorb over a given period of time,
respectively. relative to the emissions of 1 ton of carbon dioxide (CO2).

641, First Floor, Dr. Mukherjee Nagar, Delhi-110009 2


Contacts% 011-47532596, (+91) 8448485517
E-mail: helpline@groupdrishti.com, Web: www.drishtiias.com Facebook: facebook.com/drishtithevisionfoundation
Copyright – Drishti The Vision Foundation
https://telegram.me/pdf4exams https://telegram.me/ias201819

zzThe larger the GWP, the more that a given gas warms the 9. (c)
Earth compared to CO2 over that time period. Exp.: Goods and Services Tax Appellate Tribunal (GSTAT)
zzThe time period usually used for GWPs is 100 years. zzThe National Bench of the Appellate Tribunal shall be
zzThe correct ascending order as per Global Warming situated at New Delhi. GSTAT shall be presided over by
Potential is – Carbon dioxide < Methane < Nitrous its President and shall consist of one Technical Member
oxide < Chlorofluorocarbons. Hence, Option (d) is (Centre) and one Technical Member (State).
correct.
zzGoods and Services Tax Appellate Tribunal is the forum
7. (c) of second appeal in GST laws and the first common forum
Exp.: Horizon 2020 is the biggest EU Research and Innovation of dispute resolution between Centre and States. Hence,
programme ever with nearly €80 billion funding available statement 2 is correct.
over 7 years (2014 to 2020) – in addition to the private zzThe appeals against the orders in first appeals issued by
investment that this money will attract. It promises more the Appellate Authorities under the Central and State GST
breakthroughs, discoveries and world-firsts by taking Acts lie before the GST Appellate Tribunal, which is
great ideas from the lab to the market. Hence, option (c) common under the Central as well as State GST Acts.
is correct.
Hence, statement 1 is correct.
zzHorizon 2020 is the financial instrument implementing
the Innovation Union, a Europe 2020 flagship initiative zzBeing a common forum, GST Appellate Tribunal will
aimed at securing Europe's global competitiveness. ensure that there is uniformity in redressal of disputes
zzBy coupling research and innovation, Horizon 2020 is
arising under GST, and therefore, in implementation of
helping to achieve this with its emphasis on excellent GST across the country.
science, industrial leadership and tackling societal 10. (d)
challenges. The goal is to ensure Europe produces world-
class science, removes barriers to innovation and makes Exp.: Input Tax Credit (ITC) is a mechanism to avoid cascading
it easier for the public and private sectors to work together of taxes. Cascading of taxes, in simple language, is ‘tax
in delivering innovation. on tax’.
zzITC is not available in some cases as mentioned in section
8. (b) 17(5) of CGST Act, 2017. Some of them are as follows:
Exp.: The National Anti-profiteering Authority (NAA) is the (a) Motor vehicles and other conveyances except under
institutional mechanism under GST law to check the
specified circumstances.
unfair profit-making activities by the trading community.
Hence, Statement 1 is NOT Correct. (b) Goods and / or services provided in relation to:
zzThe Authority’s core function: i. Food and beverages, outdoor catering, beauty
treatment, health services, cosmetic and
„„To ensure that the benefits of the reduction in GST
rates on goods and services made by GST Council plastic surgery, except under specified
and proportional change in the Input tax credit is circumstances;
passed on to the ultimate consumers and recipient ii. membership of a club, health and fitness center.
respectively by way of reduction in the prices by the iii. Rent-a-cab, life insurance, health insurance
suppliers. except where it is obligatory for an employer
„„ Traders are not realizing unfair profit by charging under any law.
high price from consumers in the name of GST. iv. travel benefits extended to employees on
Hence, Statement 2 is Correct. vacation such as leave or home travel
zzThe responsibility of NAA is to examine and check such concession. Hence, option (d) is correct.
profiteering activities and recommend punitive actions
including cancellation of registration. 11. (d)
zzThe chairman, NAA along with 4 Technical members Exp.: An e-way bill is a document required to be carried by a
with help of the Standing Committee, Screening person in charge of the conveyance carrying any
Committee in every state and the Directorate General of consignment of goods of value exceeding fifty thousand
Safeguards in the Central Board of Excise & Customs rupees as mandated by the Government in terms of
(CBEC), will work together on the anti-profiteering front. Section 68 of the Goods and Services Tax It is generated
641] çFke ry] eq[kthZ uxj] fnYyh&9 3
nwjHkk"k % 011-47532596, (+91) 8448485517
bZ&esy % helpline@groupdrishti.com, osclkbV % www.drishtiias.com isQlcqd % facebook.com/drishtithevisionfoundation
Copyright – Drishti The Vision Foundation
https://telegram.me/pdf4exams https://telegram.me/ias201819

from the GST Common Portal for e-Way bill system by zzMany importers and exporters in the country, for instance,
the registered persons or transporters who cause may be locked into binding agreements that could force
movement of goods of consignment before commencement them to buy or sell a certain number of goods despite the
of such movement. unfavourable exchange rate of the currency.
zzThe consignor or consignee, as a registered person or a Beveridge curve
transporter of the goods can generate the e-way bill. The
unregistered transporter can enroll on the common portal zzRefers to a graphical representation that shows the
and generate the e-way bill for movement of goods for relationship between the unemployment rate (on the
his clients. Any person can also enroll and generate the horizontal axis) and the job vacancy rate (on the vertical
e-way bill for movement of goods for his/her own use. axis) in an economy.
Hence, statement 1 is NOT correct. zzIt is named after British economist William Beveridge.
zzThe validity period of the EWB is calculated based on The Beveridge curve usually slopes downwards because
the ‘approx. distance’ entered while generating the EWB. times when there is high job vacancy in an economy are
For every 100 Kms one day is a validity period for EWB also marked by relatively low unemployment since
as per rule and for part of 100 KM one more day is added. companies may actually be actively looking to hire new
For ex. If approx. distance is 310 Kms then validity period people. By the same logic, a low job vacancy rate usually
is 3+1 days. For movement of Over Dimensional Cargo
corresponds with high unemployment as companies may
(ODC), the validity is one day for every 20 KM (instead
not be looking to hire many people in new jobs.
of 100 KM)and for every 20 KM or part thereof one more
day is added. Please refer relevant rules for details. Hence, 14. (c)
statement 2 is NOT correct.
Exp.: The Union Cabinet has approved the Agriculture Export
12. (a) Policy, 2018.
Exp.: Union Ministry of Commerce & Industry launched Niryat Objectives of the Agriculture Export Policy are as
Mitra – mobile App under:
zzThe app developed by the Federation of Indian Export zzTo double agricultural exports from present ~US$ 30+
Organisations (FIEO) is available both on Android and Billion to ~US$ 60+ Billion by 2022 and reach US$ 100
on IOS platforms. It provides wide range of information Billion in the next few years thereafter, with a stable trade
required to undertake international trade right from the policy regime.
policy provisions for export and import, applicable GST
zzTo diversify our export basket, destinations and boost
rate, available export incentives, tariff, preferential tariff,
high value and value added agricultural exports including
market access requirements – SPS and TBT measures.
The most interesting part is that all the information is focus on perishables. Hence, statement 1 is correct.
available at tariff line. The app works internally to map zzTo promote novel, indigenous, organic, ethnic, traditional
the ITC HS code of other countries with that of India and and non-traditional Agri products exports.
provides all the required data without the users bothering zzTo provide an institutional mechanism for pursuing
about the HS code of any country. Presently the app comes market access, tackling barriers and deal with sanitary
with the data of 87 countries. Hence, option (a) is correct. and phyto-sanitary issues. Hence, statement 2 is correct.
13. (b) zzTo strive to double India’s share in world agri-exports by
Exp.: J-curve effect integrating with global value chain at the earliest.
zzRefers to a phenomenon wherein the trade balance of a zzEnable farmers to get benefit of export opportunities in
country worsens following the depreciation of its currency overseas market.
before it improves. Hence, statement 1 is NOT correct.
15. (a)
zzGenerally, any depreciation in the value of a currency is
expected to improve the economy’s overall trade balance Exp.: The Code creates time-bound processes for insolvency
by encouraging exports and discouraging imports. resolution of companies and individuals. These processes
However, this may not happen immediately due to some will be completed within 180 days. If insolvency cannot
other frictions within the economy. Hence, statement 2 be resolved, the assets of the borrowers may be sold to
is correct. repay creditors. Hence, statement 1 is correct.

641, First Floor, Dr. Mukherjee Nagar, Delhi-110009 4


Contacts% 011-47532596, (+91) 8448485517
E-mail: helpline@groupdrishti.com, Web: www.drishtiias.com Facebook: facebook.com/drishtithevisionfoundation
Copyright – Drishti The Vision Foundation
https://telegram.me/pdf4exams https://telegram.me/ias201819

zzThe resolution processes will be conducted by licensed zzThe panel would also suggest an adequate level of risk
insolvency professionals (IPs). These IPs will be members provisioning that the RBI needs to maintain, and to
of insolvency professional agencies (IPAs). IPAs will also determine whether it is holding provisions, reserves and
furnish performance bonds equal to the assets of a buffers in surplus or deficit of the required level.
company under insolvency resolution.
zzThe committee would also propose a suitable profit
zzInformation utilities (IUs) will be established to collect,
distribution policy taking into account all the likely
collate and disseminate financial information to facilitate
situations of the RBI, including holding more provisions
insolvency resolution. Hence, statement 2 is correct.
than required and the RBI holding less provisions than
zzThe National Company Law Tribunal (NCLT) will
required, etc.
adjudicate insolvency resolution for companies. The Debt
Recovery Tribunal (DRT) will adjudicate insolvency 18. (d)
resolution for individuals. Hence, statement 3 is NOT
correct. Exp.: The Integrated Development of Wildlife Habitats
(IDWH) is a Centrally Sponsored Scheme launched
zzThe Insolvency and Bankruptcy Board of India will be
set up to regulate functioning of IPs, IPAs and IUs. during the 11th Plan period to provide technical and
financial assistance to States/UTs for protection of
16. (a) wildlife habitat.
Exp.: The Fugitive Economic Offenders Bill, 2018 allows for zzThe activities covered under the scheme include the staff
a person to be declared as a fugitive economic offender development and capacity building, wildlife research and
(FEO) if: evaluation, anti-poaching activities, wildlife veterinary
(i) An arrest warrant has been issued against him for care, addressing man-animal conflicts and promoting
any specified offences where the value involved is eco-tourism.
over Rs 100 crore, and
zzFinancial assistance is also provided to States for
(ii) He has left the country and refuses to return to face
relocation of communities from within protected areas to
prosecution.
other areas.
zzTo declare a person an FEO, an application will be filed
in a Special Court (designated under the Prevention of The scheme has following three components:
Money-Laundering Act, 2002) containing details of the 1. Support to Protected Areas (National Parks, Wildlife
properties to be confiscated, and any information about Sanctuaries, Conservation Reserves and Community
the person’s whereabouts. Hence, option (a) is correct.
Reserves)
zzThe Special Court will require the person to appear at a
2. Protection of Wildlife outside Protected Areas
specified place at least six weeks from issue of notice.
Proceedings will be terminated if the person appears. 3. Recovery programmes for saving critically
endangered species and habitats. Hence, option (d)
17. (b) is correct.
Exp.: The Reserve Bank of India has formed an expert zzThe scheme consists of centrally sponsored scheme of
committee under former governor Bimal Jalan to decide Project Tiger (CSS-PT), Development of Wildlife
the appropriate level of reserves that the regulator should Habitats (CSS-DWH), and Project Elephant (CSS-PE).
hold. Hence, option (b) is correct.
zzThe committee would submit its report within 90 days 19. (c)
from the date of its first meeting. The terms of reference Exp.: The BioCarbon Fund Initiative for Sustainable Forest
of the committee would be to review status, need and
Landscapes (ISFL) is a multilateral fund, supported by
justification of various provisions, reserves and buffers
donor governments and managed by the World Bank.
presently provided for by the RBI, keeping in mind ‘public
policy mandate of the RBI, including financial stability Hence, statement 2 is correct.
considerations.’ zzIt promotes reducing greenhouse gas emissions from the
zzThe committee will also review best practices followed
land sector, from deforestation and forest degradation in
by the central banks globally in making assessment and developing countries (REDD+), and from sustainable
provisions for risks, to which central bank balance sheets agriculture, as well as smarter land-use planning, policies
are subjected. and practices. Hence, statement 1 is correct.

641] çFke ry] eq[kthZ uxj] fnYyh&9 5


nwjHkk"k % 011-47532596, (+91) 8448485517
bZ&esy % helpline@groupdrishti.com, osclkbV % www.drishtiias.com isQlcqd % facebook.com/drishtithevisionfoundation
Copyright – Drishti The Vision Foundation
https://telegram.me/pdf4exams https://telegram.me/ias201819

zzISFL has a geographically diverse portfolio of large-scale zzAdditionally, during an agriculture crop year, when there
programs that can have significant impact and transform is projected over supply of food grains as anticipated by
rural areas by protecting forests, restoring degraded lands, the Ministry of Agriculture & Farmers Welfare, the policy
enhancing agricultural productivity, and by improving allows conversion of surplus quantities of food grains to
livelihoods and local environments. ethanol, based on the approval of National Biofuel
Coordination Committee.
20. (d)
22. (a)
Exp.: The Climate and Clean Air Coalition is a voluntary
partnership of governments, intergovernmental Exp.: The New Delhi Declaration on Asian Rhinos 2019
zzIndia in collaboration with Bhutan, Nepal, Indonesia and
organizations, businesses, scientific institutions and civil
Malaysia has signed ‘The New Delhi Declaration on
society organizations committed to improving air quality
Asian Rhinos 2019’ for the conservation and protection
and protecting the climate through actions to reduce
of the species at the recently held Second Asian Rhino
short-lived climate pollutants. Hence, statement 1 and
Range Countries meeting in New Delhi.
2 are NOT correct.
zzThe declaration was signed to conserve and review the
zzIn 2012, the governments of Bangladesh, Canada, Ghana,
population of the Greater one-horned (GOHR), Javan and
Mexico, Sweden and the United States, along with the
Sumatran rhinos every four years to reassess the need for
United Nations Environment Programme (UNEP), came
joint actions to secure their future. Hence, statement 1
together to initiate efforts to treat short-lived climate
is correct and 2 is NOT correct.
pollutants as an urgent and collective challenge.
zzThe meet was organised by the Environment, Forest and
zzTogether, they formed the Climate & Clean Air Coalition
Climate Change Ministry in collaboration with IUCN
to support fast action and deliver benefits on several fronts Asian Rhino Specialist Group, WWF- India and
at once: climate, public health, energy efficiency, and food Aaranyak.
security.
zz1st Asian Rhino Range States meeting was held at Bandar
zzToday, the Coalition brings together hundreds of Lampung, Indonesia in October 2013. Asian Rhino Range
experienced and influential stakeholders from around the Countries include Bhutan, India, Indonesia, Malaysia,
world to leverage high-level engagement and catalyse and Nepal.
concrete actions in both the public and private sector.
Extra Information:
21. (a) zzRhino vision 2020 was launched in 2005 by the Assam
Forest Department in collaboration with International
Exp.: With the objective of ensuring adequate and sustained
Rhino Foundation (IRF) and WWF-India. The program’s
availability of domestic feedstock for biofuel production,
vision is to expand the number of GOHR to 3,000
increasing Farmers Income, Import Reduction, individuals spread over seven protected areas by the year
Employment Generation and Waste to Wealth Creation, 2020. Rhinos presently inhabit four areas in Assam:
the Government has notified the National Policy on Kaziranga, Orang, Pabitora, and Manas. IRV 2020 is
Biofuels-2018. targeting Burachapori Wildlife Sanctuary, Laokhowa
zzThe policy categorizes biofuels as: Wildlife Sanctuary and DibruSaikhowa NP for future
„„Basic Biofuels, i.e.,such as bio ethanol & biodiesel translocations.
„„Advanced Biofuels such as Second Generation (2G) 23. (c)
ethanol, bio-CNG, Third Generation Biofuels, etc.
Exp.: National Air Quality Monitoring Programme
zzTo enable extension of appropriate financial and fiscal
zzCentral Pollution Control Board is executing a nation-
incentives under each category. Hence, option (a) is wide programme of ambient air quality monitoring known
correct. as National Air Quality Monitoring Programme (NAMP).
zzWith an objective of increasing production of ethanol, Hence, statement 1 is correct
this Policy allows production of ethanol from damaged zzThe NAMP consists of a network of 731 operating stations
food grains like wheat, broken rice, etc. which are unfit covering 312 cities/towns in 29 states and 6 Union
for human consumption. Territories of the country.

641, First Floor, Dr. Mukherjee Nagar, Delhi-110009 6


Contacts% 011-47532596, (+91) 8448485517
E-mail: helpline@groupdrishti.com, Web: www.drishtiias.com Facebook: facebook.com/drishtithevisionfoundation
Copyright – Drishti The Vision Foundation
https://telegram.me/pdf4exams https://telegram.me/ias201819

zzThe objectives of the NAMP are to determine the status is paramount to improve scientific understanding about
and trends of ambient air quality; to ascertain whether the climate change and to generate new knowledge about
prescribed ambient air quality standards are violated; to climate change impacts in the region. Hence, statement
Identify Non-attainment Cities; to obtain the knowledge 1 is correct.
and understanding necessary for developing preventive zzPhase 1 of the programme was implemented from 2012
and corrective measures and to understand the natural to 2015. Building on the results and experience from
cleansing process undergoing in the environment through Phase 1, IHCAP as part of Phase 2 (2016-19) will continue
pollution dilution, dispersion, wind based movement, dry to work at both the national and sub-national levels.
deposition, precipitation and chemical transformation of
26. (d)
pollutants generated.
Exp.: International Food Policy Research Institute
zzUnder NAMP, four air pollutants viz ., Sulphur Dioxide
zzThe International Food Policy Research Institute (IFPRI)
(SO2), Oxides of Nitrogen as NO2, Respirable Suspended
provides research-based policy solutions to sustainably
Particulate Matter (RSPM / PM10) and Fine Particulate
reduce poverty and end hunger and malnutrition in
Matter (PM2.5) have been identified for regular
developing countries.
monitoring at all the locations. Hence, statement 2 is
zzEstablished in 1975, IFPRI currently has more than 600
correct.
employees working in over 50 countries.
zzThe monitoring is being carried out with the help of Central
zzIFPRI’s Strategy Refresh 2018–2020 builds on the strong
Pollution Control Board; State Pollution Control Boards;
base of work developed under the Institute’s 2013–2018
Pollution Control Committees; National Environmental
strategy and focuses on five strategic research areas:
Engineering Research Institute (NEERI), Nagpur.
„„Fostering climate-resilient and sustainable food supply
24. (d) „„Promoting healthy diets and nutrition for all
Exp.: Global Energy and Status Report 2018 „„Transforming agriculture and rural economies
zzChina, India, and the United States accounted for 85% of „„Strengthening institutions and governance
the net increase in emissions. Renewables increased by
„„Building inclusive and efficient markets, trade systems
4% in 2018, accounting for almost one-quarter of global and food industry. Hence, option (d) is correct.
energy demand growth. Hence, statement 1 is correct.
zzIndia comes in the category of IEA Association Countries, 27. (b)
not in IEA Member Countries. Hence, statement 3 is Exp.: Blue flag standards certification programme:
NOT correct. zzIt was established by Copenhagen-based Foundation for
zzThe onus of publishing the Global Energy and Status Environmental Education (FEE) an international non-
Report is on International Energy Agency. governmental, non-profit organisation in 1985. Hence,
statement 1 is NOT correct.
zzGrowth in India was led by coal (for power generation)
and oil (for transport), the first and second biggest zzThe Blue Flag Programme aims to promote sustainable
contributors to energy demand growth, respectively. development in freshwater and marine areas, challenging
Hence, statement 2 is correct. local authorities to achieve high standards in the four
broad categories of water quality, environmental
25. (a) management, environmental education and safety. Hence,
Exp.: Indian Himalayas Climate Adaptation Programme: Statement 2 is correct.
zzIHCAP is a project of the Swiss Agency for Development zzThe Blue Flag certification tag is given to beaches that
and Cooperation (SDC), and is being implemented in are environment-friendly and clean, equipped with
partnership with the Department of Science and amenities of international standards for tourists and have
Technology (DST), Government of India. Hence, facilities for studying environmental impact around the
statement 2 is correct while statement 3 is NOT beach.
correct. zzThe Chandrabhaga beach on the Konark coast of Odisha
zzThe vulnerability of the Himalayan region to climate has become the Asia’s first beach to get the Blue Flag
change and the dependence of a vast population on it, it certification. Hence, Statement 3 is correct.

641] çFke ry] eq[kthZ uxj] fnYyh&9 7
nwjHkk"k % 011-47532596, (+91) 8448485517
bZ&esy % helpline@groupdrishti.com, osclkbV % www.drishtiias.com isQlcqd % facebook.com/drishtithevisionfoundation
Copyright – Drishti The Vision Foundation
https://telegram.me/pdf4exams https://telegram.me/ias201819

28. (d) 30. (c)


Exp.: The Republic of Congo, Democratic Republic of Congo Exp.: South Asia Wildlife Enforcement Network (SAWEN):
and Indonesia signed the Brazzaville Declaration at the zzIt is an inter-governmental wildlife law enforcement
third meeting of the Global Peatland Initiative to conserve support body of South Asian countries namely -
peatlands in the Congo basin. The Cuvette Centrale Afghanistan, Bangladesh, Bhutan, India, Maldives,
peatlands of the Congo are globally important asset in Nepal, Pakistan and Sri Lanka.
the fight against climate change, biodiversity loss and zzIt promotes regional cooperation to combat wildlife crime
social instability. in South Asia. It focuses on policy harmonization,
zzIt promotes better management and conservation of the institutional capacity strengthening through knowledge
peatlands, which represent one of the biggest carbon stores and intelligence sharing, and collaboration with regional
on the planet, for climate mitigation and other benefits. and international partners to enhance wildlife law
zzThe Cuvette Centrale peatland complex extends across The enforcement in the member countries. Hence, (c) is the
Republic of Congo and the Democratic Republic of Congo. correct answer.
It is estimated to hold about 30 billion tonnes of carbon
31. (b)
and cover over 145,500 square kilometres. These relatively
undisturbed peatland forests, which have been inhabited Exp.: Many of India’s aquifers are composed of clay, silt and
for more than 50,000 years, are home to unique species of gravel carried down from Himalayan weathering by
plants and animals. Hence, (d) is the correct answer. streams or uranium-rich granitic rocks.
Note- Importance of peatlands zzWhen over-pumping of these aquifers’ groundwater
occurs and their water levels decline, it induces oxidation
1. Peatlands are the largest natural terrestrial carbon conditions that, in turn, enhance uranium enrichment in
store and contains more carbon than any other the shallow groundwater that remains.
vegetation type, including the world’s forests. zzThe primary source of uranium is geogenic (naturally
2. Peatlands provide safe drinking water, minimise the occurring), anthropogenic (human caused) factors such
risk of flood and drought. as groundwater table decline and nitrate pollution further
3. Peatlands are critical for preserving global enhances uranium contamination. Hence, Statement 1
biodiversity. is NOT correct.
29. (a) zzUranium has yet not been included in the list of
contaminants monitored under the Bureau of Indian
Exp.: Atmosphere & Climate Research-Modelling Standards’ Drinking Water Specifications. Hence,
Observing Systems & Services (ACROSS) scheme: Statement 2 is correct.
zzACROSS scheme is related to the atmospheric science
zzUranium-laced water if consumed more than permissible
programs of the Ministry of Earth Sciences (MoES) and
level(30 micrograms of uranium per litre for India) for
will be implemented by the its institutes namely India
long time then it can cause thyroid cancer, blood cancer,
Meteorological Department(IMD), Indian Institute of
depression and other serious ailments.
Tropical Meteorology (IITM), National Centre for
Medium Range Weather Forecasting (NCMRWF), and 32. (c)
Indian National Centre for Ocean Information Service Exp.: The polar vortex is a large area of low pressure and cold
(INCOIS). Hence, Statement 1 is NOT correct. air surrounding both of the Earth’s poles. Hence,
zzIt will addresses different aspects of weather and climate statement 2 is NOT correct.
services, which includes warnings for cyclone, storm zzIt always exists near the poles, but weakens in summer
surges, heat waves, thunderstorms etc. Hence, Statement and strengthens in winter. Hence, statement 3 is correct.
3 is correct. zzThe term ‘vortex’ refers to the counter-clockwise flow of
zzIt will ensure transfer of benefits to various services like air that helps keep the colder air near the Poles. Hence,
public weather service, disaster management, agro- statement 1 is correct.
meteorological services, aviation services, environmental zzIn winter, the polar vortex sometimes becomes less stable
monitoring services, hydro-meteorological services, and expands. Many times during winter in the northern
climate services, tourism, pilgrimage, power generation, hemisphere, the [north] polar vortex will expand, sending
water management, sports and adventure. Hence, cold air southward with the jet stream, this is called a
Statement 2 is correct. polar vortex event.
641, First Floor, Dr. Mukherjee Nagar, Delhi-110009 8
Contacts% 011-47532596, (+91) 8448485517
E-mail: helpline@groupdrishti.com, Web: www.drishtiias.com Facebook: facebook.com/drishtithevisionfoundation
Copyright – Drishti The Vision Foundation
https://telegram.me/pdf4exams https://telegram.me/ias201819

zzNormally, when the vortex is strong and healthy, it helps Factors affecting ocean salinity are mentioned below:
keep a current of air known as the jet stream travelling zzThe salinity of water in the surface layer of oceans
around the globe in a pretty circular path. This current depends mainly on evaporation and precipitation. Hence,
keeps the cold air up north and the warm air down south. statement 1 is correct.
But without that strong low-pressure system, the jet zzSurface salinity is greatly influenced in coastal regions
stream doesn’t have much to keep it in line. It becomes by the fresh water flow from rivers, and in Polar Regions
wavy and rambling. by the processes of freezing and thawing of ice.
zzMany times during winter in the northern hemisphere, zzWind, also influences salinity of an area by transferring
the polar vortex will expand, sending cold air southward water to other areas.
with the jet stream. zzThe ocean currents contribute to the salinity variations.
Salinity, temperature and density of water are interrelated.
33. (b) Hence, any change in the temperature or density
Exp.: The factors which affect the distribution of temperature influences the salinity of water in an area. Hence ,
of ocean water are : statement 2 is correct.
zzPrevailing winds: the winds blowing from the land zzThe average salinity of the Indian Ocean is 35 o/oo. The
towards the oceans drive warm surface water away from low salinity trend is observed in the Bay of Bengal due
the coast resulting in the upwelling of cold water from to influx of river water. On the contrary, the Arabian Sea
below. It results in the longitudinal variation in the shows higher salinity due to high evaporation and low
temperature. Contrary to this, the onshore winds pile up influx of fresh water. Hence, statement 3 is NOT correct.
warm water near the coast and this raises the temperature. 35. (a)
zzUnequal distribution of land and water: the oceans in
Exp.: Major currents in Indian oceans:
the northern hemisphere receive more heat due to their
zzWest Australian current
contact with larger extent of land than the oceans in the zzAuglhas current
southern hemisphere. zzWest wind drift
zzOcean currents: warm ocean currents raise the zzEquatorial counter current
temperature in cold areas while the cold currents decrease zzSouth Equatorial current
the temperature in warm ocean areas. Gulf stream (warm Major currents in Pacific oceans:
current) raises the temperature near the eastern coast of zzAlaska current
North America and the West Coast of Europe while the zzCalifornia current
Labrador current (cold current) lowers the temperature zzN. pacific drift
near the north-east coast of North America. zzNorth Equatorial current
zzLatitude: the temperature of surface water decreases zzEq. counter current
from the equator towards the poles because the amount zzSouth Equatorial current
of insolation decreases poleward. While longitude does zzHumboldt (peru) current
not have any effect on the distribution of temperature of zzWest wind drift
ocean water. Hence, option (b) is correct. zzOyashio current
zzKuroshio current
zzAll these factors influence the temperature of the ocean
zzWest wind drift
currents locally. The enclosed seas in the low latitudes
Major currents in Atlantic oceans:
record relatively higher temperature than the open seas;
zzLabrador current
whereas the enclosed seas in the high latitudes have lower
zzNorth atlantic drift
temperature than the open seas.
zzGulf stream current
34. (a) zzCanaries current
zzN. equatorial current
Exp.: All waters in nature, whether rain water or ocean water,
zzEq. counter current
contain dissolved mineral salts. Salinity is the term used zzSouth equatorial current
to define the total content of dissolved salts in sea water. zzBenguela current
It is calculated as the amount of salt (in gm) dissolved in zzBrazilian current
1,000 gm (1 kg) of seawater. It is usually expressed as zzFalkland current
parts per thousand (o/oo ) or ppt. zzWest wind drift

641] çFke ry] eq[kthZ uxj] fnYyh&9 9


nwjHkk"k % 011-47532596, (+91) 8448485517
bZ&esy % helpline@groupdrishti.com, osclkbV % www.drishtiias.com isQlcqd % facebook.com/drishtithevisionfoundation
Copyright – Drishti The Vision Foundation
https://telegram.me/pdf4exams https://telegram.me/ias201819

36. (a) 39. (c)


Exp.: The troposphere is the lowermost layer of the atmosphere. Exp.: The river Ganga has a length of 2,525 km. It is shared
Its average height is 13 km and extends roughly to a height by Uttarakhand (110 km) and Uttar Pradesh (1,450 km),
of 8 km near the poles and about 18 km at the equator. Bihar (445 km) and West Bengal (520 km).
zzThickness of the troposphere is greatest at the equator zzThe Ganga basin covers about 8.6 lakh sq. km area in
because heat is transported to great heights by strong India alone.
convectional currents. Hence, statement 1 is correct. zzThe Ganga river system is the largest in India having a
zzThis layer contains dust particles and water vapour. All number of perennial and non-perennial rivers originating
changes in climate and weather take place in this layer. in the Himalayas in the north and the Peninsula in the
zzThe temperature in this layer decreases at the rate of 1°C south, respectively. Hence, statement 1 is correct.
for every 165m of height. This is the most important layer zzThe Mahananda is important tributary of the Ganga
for all biological activity. rising in the Darjeeling hills. It joins the Ganga as its last
zzThe stratosphere is found above the tropopause and left bank tributary in West Bengal. Hence, statement
extends up to a height of 50 km. One important feature 2 is correct.
of the stratosphere is that it contains the ozone layer. This zzThe important left bank tributaries are the Ramganga, the
layer absorbs ultra-violet radiation and shields life on the Gomati, the Ghaghara, the Gandak, the Kosi and the
earth from intense, harmful form of energy. Hence Mahananda, etc.
,statement 2 is NOT correct.
zzThe right bank tributaries of the Ganga are The Yamuna,
zzThe mesosphere lies above the stratosphere, which The Son, etc.
extends up to a height of 80 km. In this layer, once again,
temperature starts decreasing with the increase in altitude 40. (a)
and reaches up to minus 100°C at the height of 80 km. Exp.: Edward Robert Bulwer-Lytton was viceroy of India
37. (b) between 1876 and 1880. He was appointed by Prime
Minister Benjamin Disraeli, a Conservative, at a time of
Exp.: The north Indian rivers originating from the Himalayas intense competition between Britain and Russia over
are perennial as they are fed by glaciers through snow control of Central Asia. Important events that took place
melt and also receive rainfall water during rainy season. under Lytton are:
zzThe rivers of South India do not originate from glaciers
zzDelhi Durbar of 1877: To proclaim the imperial title by
and their flow pattern witnesses fluctuations. The flow
the queen. It coincided with a severe famine and instead
increases considerably during monsoon rains. Thus, the
of sending relief to the afflicted money was spent on an
regime of the rivers of South India is controlled by rainfall
extravagant durbar.
which also varies from one part of the Peninsular plateau
to the other. Hence, statement 1 is NOT correct. zzArms Act 1878: It sought to impose restrictions on the
possession of arms by Indians while europeans were
zzThe Narmada and The Tapi flow in trough faults and fill
allowed to keep arms.
the original cracks with their detritus materials. Hence,
there is a lack of alluvial and deltaic deposits in these zzVernacular Press Act 1878: It sought to muzzle the
rivers. growing criticism of the government policies by imposing
restrictions on regional language news papers.
zzPeninsular rivers are characterised by fixed course,
absence of meanders and non perennial flow of water. zzReduction in the age for ICS: In 1878 a regulation was
The Nature of these rivers is smaller, fixed course with passed to reduce the age for appearing in the ICS from
well-adjusted valleys. Hence , statement 2 is correct. 21 to 19. Hence, option (a) is correct.

38. (d) 41. (a)


Exp.: ‘HELINA’ is indigenously developed Anti-Tank Guided Exp.: The process of imperial expansion and consolidation of
Missile. British paramountcy was carried on by the Company
zzThe Missile is guided by an Infrared Imaging Seeker during the 1757-1857 period through a two-fold method:
operating in the Lock on Before Launch mode. It is one (a) policy of annexation by conquest or war;
of the most advanced Anti-Tank Weapons in the world. (b) policy of annexation by diplomacy and
Hence, option (d) is correct. administrative mechanisms.
641, First Floor, Dr. Mukherjee Nagar, Delhi-110009 10
Contacts% 011-47532596, (+91) 8448485517
E-mail: helpline@groupdrishti.com, Web: www.drishtiias.com Facebook: facebook.com/drishtithevisionfoundation
Copyright – Drishti The Vision Foundation
https://telegram.me/pdf4exams https://telegram.me/ias201819

zzWarren Hastings’ ‘ring-fence’ policy: It was the policy 4. It laid stress on female and vocational education,
of defence of neighbours’ frontiers for safeguarding their and on teachers’ training.
own territories. Hence, pair 1 is correctly matched. 5. It laid down that the education imparted in
zzWellesley’s system of ‘subsidiary alliance’: Under the government institutions should be secular.
system, the allying Indian state’s ruler was compelled to 6. It recommended a system of grants-in-aid to
accept the permanent stationing of a British force within encourage private enterprise. Hence, option (b) is
his territory and to pay a subsidy for its maintenance. correct.
zzAlso, the Indian ruler had to agree to the posting of a 43. (a)
British resident in his court.
Exp.: The British government’s decision to partition Bengal
zzUnder the system, the Indian ruler could not employ any
had been made public in December 1903.
European in his service without the prior approval of the
On August 7, 1905,with the passage of the Boycott
British. Nor could he negotiate with any other Indian ruler
Resolution in a massive meeting held in the Calcutta
without consulting the governor-general. In return for all
Townhall, the formal proclamation of Swadeshi
this, the British would defend the ruler from his enemies
Movement was made. After this, the leaders dispersed
and adopt a policy of non- interference in the internal
to other parts of Bengal to propagate the message of
matters of the allied state. Hence, pair 2 is NOT correctly
boycott of Manchester cloth and Liverpool salt.
matched.
October 16, 1905, the day the partition formally came
zzDalhousie’s ‘doctrine of lapse’: The doctrine stated that
he calcutta session of indian national
into force. T
the adopted son could be the heir to his foster father’s
congress was held on 1906(Dec. 26-29). The Congress
private property, but not the state; it was for the paramount
split at Surat came in December 1907.
power (the British) to decide whether to bestow the state
on the adopted son or to annex it. The doctrine was stated Hence, option (a) is correct.
to be based on Hindu law and Indian customs, but Hindu 44. (c)
law seemed to be somewhat inconclusive on this point,
Exp.: The All India Trade Union Congress (AITUC) was
and the instances of an Indian sovereign annexing the
founded on October 31, 1920. The Indian National
state of his vassal on account of ‘lapse’ (i.e., leaving no
Congress president for the year, Lala Lajpat Rai, was
issue as heir) were rather rare. Maharaja Ranjit Singh had
elected as the first president of AITUC and Dewan
annexed a few of his feudatory principalities on account
Chaman Lal as the first general secretary. Lajpat Rai was
of ‘lapse’. Hence, pair 3 is NOT correctly matched.
the first to link capitalism with imperialism— “imperialism
42. (b) and militarism are the twin children of capitalism”. The
Exp.: In 1854, Charles Wood prepared a despatch on an prominent Congress and swarajist leader C.R. Das
educational system for India. Considered the “Magna presided over the third and the fourth sessions of the
Carta of English Education in India”, this document was AITUC.
the first comprehensive plan for the spread of education 45. (d)
in India.
Exp.: Trade Union Act, 1926
1. It asked the government of India to assume
zzRecognised trade unions as legal associations.
responsibility for education of the masses, thus
repudiating the ‘downward filtration theory’. zzLaid down conditions for registration and regulation of
trade union activities;
2. It systematised the hierarchy from vernacular
primary schools in villages at bottom, followed by zzSecured immunity, both civil and criminal, for trade
Anglo-Vernacular High Schools and an affiliated unions from prosecution for legitimate activities, but put
college at the district level, and affiliating some restrictions on their political activities.
universities in the presidency towns of Calcutta, Hence, statement 1 is NOT correct.
Bombay and Madras. zzThe whole of 1928 witnessed unprecedented industrial
3. It recommended English as the medium of unrest. This period also saw the creation of various
instruction for higher studies and vernaculars at communist groups, with leaders like S.A. Dange,
school level. Muzaffar Ahmed, P.C. Joshi, Sohan Singh Joshi etc.
641] çFke ry] eq[kthZ uxj] fnYyh&9 11
nwjHkk"k % 011-47532596, (+91) 8448485517
bZ&esy % helpline@groupdrishti.com, osclkbV % www.drishtiias.com isQlcqd % facebook.com/drishtithevisionfoundation
Copyright – Drishti The Vision Foundation
https://telegram.me/pdf4exams https://telegram.me/ias201819

Alarmed at the increasing strength of the trade union Nation. This will further help them to appreciate what
movement under extremist influence, the government they are being taught in the school and its real application
resorted to legislative restrictions. It passed the Public in Space Science & Technology. ISRO has chalked out
Safety Ordinance (PSO), 1929 and the Trade Disputes this programme to “Catch them young”.
Act (TDA), 1929. zzThe programme will be of around two weeks duration
zzIn March 1929, the Government arrested 31 labour leaders during summer holidays.
in the Meerut Conspiracy case based on the PSO and zzIt is proposed to select 3 students each from each State/
TDA, and the three-and-a-half-year trial resulted in the Union Territory to participate in this programme every
conviction of Muzaffar Ahmed, S.A. Dange, Joglekar, year covering CBSE, ICSE and State syllabus. Those who
Philip Spratt, Ben Bradley, Shaukat Usmani and others. have finished 8th standard and currently studying in 9th
The trial got worldwide publicity but weakened the standard will be eligible for the programme. Hence,
working class movement. Hence, statement 2 is NOT statement 2 is NOT correct.
correct.
48. (b)
46. (c) Exp.: To cater to the needs and aspirations of the youth, the
Exp.: Pradhan Mantri Shram Yogi Mandhan (PM- SYM) National Cooperative Development Corporation (NCDC)
Yojana is the one of the world’s largest pension scheme. under the Ministry of Agriculture and Farmers’
zzThe unorganised workers whose monthly income is Rs Welfare has come up with a youth-friendly scheme ‘Yuva
15,000/ per month or less and belong to the entry age Sahakar-Cooperative Enterprise Support and Innovation
group of 18-40 years are eligible for the scheme. Hence, Scheme” for attracting them to cooperative business
statement 1 is correct. ventures. Hence, statement 1 is NOT correct.
zzNCDC has created a dedicated fund with liberal features
zzThey should not be covered under New Pension Scheme
enabling youth to avail the scheme.
(NPS), Employees’ State Insurance Corporation (ESIC)
scheme or Employees’ Provident Fund Organisation zzThe scheme will be linked to Rs 1000 crore ‘Cooperative
(EPFO). Further, he/she should not be an income tax Start-up and Innovation Fund (CSIF)’ created by the
payer. NCDC. Hence, statement 2 is correct.
zzThe scheme is a central sector scheme administered by zzIt would have more incentives for cooperatives of North
the ministry of labour and employment and implemented Eastern region, Aspirational Districts and cooperatives
with women or SC or ST or PwD members.
through LIC and CSC eGovernance Services India Ltd.
Hence, statement 2 is NOT correct. zzThe funding for the project will be up to 80% of the project
cost for these special categories as against 70% for others.
zzPM-SYM is a voluntary and contributory pension
scheme on a 50:50 basis where prescribed age-specific zzThe scheme envisages 2% less than the applicable rate
contribution shall be made by the beneficiary and the of interest on term loan for the project cost up to Rs 3
matching contribution by the Central Government. crore including 2 years moratorium on payment of
Hence, statement 3 is correct. principal.
zzAll types of cooperatives in operation for at least one year
47. (d) are eligible. Hence, statement 3 is correct.
Exp.: Indian Space Research Organisation has launched a 49. (d)
special programme for School Children called “Young
Scientist Programme” (“YUva VIgyani KAryakram”). Exp.: An electoral bond is designed to be a bearer instrument
Hence, statement 1 is NOT correct. like a Promissory Note that is payable to the bearer on
zzThe Program is primarily aimed at imparting basic demand and free of interest.
zzThe bonds are issued in multiples of `1,000, `10,000, `1
knowledge on Space Technology, Space Science and
Space Applications to the younger ones with the intent lakh, `10 lakh and `1 crore and available at specified
branches of State Bank of India. They can be bought by
of arousing their interest in the emerging areas of Space
the donor with a KYC-compliant account. Donors can
activities.
donate the bonds to their party of choice which can then
zzThe program is thus aimed at creating awareness amongst be cashed in via the party's verified account within 15
the youngsters who are the future building blocks of our days. Hence, statement 2 is NOT correct.

641, First Floor, Dr. Mukherjee Nagar, Delhi-110009 12


Contacts% 011-47532596, (+91) 8448485517
E-mail: helpline@groupdrishti.com, Web: www.drishtiias.com Facebook: facebook.com/drishtithevisionfoundation
Copyright – Drishti The Vision Foundation
https://telegram.me/pdf4exams https://telegram.me/ias201819

zzEvery party that is registered under section 29A of the zzAfter a voter presses the button on the EVM against the
Representation of the Peoples Act, 1951 (43 of 1951) and chosen candidate, the VVPAT prints a slip containing
has secured at least one per cent of the votes polled in the name of the candidate and the election symbol and drops
most recent Lok Sabha or State election will be allotted it automatically into a sealed box. The machines give the
a verified account by the Election Commission of India. chance for the voter to verify their vote. Hence, statement
Electoral bond transactions can be made only via this 2 is NOT correct.
account. Hence, statement 1 is NOT correct. zzThe machine is placed in a glass case in a way that only
zzThe bonds will be available for purchase for a period of the voter can see it. The slip is displayed to the voter for
10 days each in the beginning of every quarter, i.e. in seven seconds after which the VVPAT machine cuts it
January, April, July and October as specified by the and drops in into the storage box with a beep. The
Central Government. An additional period of 30 days machines can be accessed, though, by the polling officials
shall be specified by the Central Government in the year and not by the voter.
of Lok Sabha elections. 52. (d)
50. (d) Exp.: The Lokpal selection committee is headed by the Prime
Exp.: In the normal course, if a person is arrested, he or she is Minister and has as its members the Lok Sabha Speaker,
guaranteed certain basic rights. These include the right leader of the opposition in the Lower House, Chief Justice
to be informed of the reason for the arrest. of India or a judge of the apex court nominated by him,
zzSection 50 of the Criminal Procedure Code (Cr.PC) and an eminent jurist who could be nominated by the
mandates that the person arrested has to be informed of President. Hence, option (d) is correct.
the grounds of arrest, and the right to bail. Sections 56 53. (c)
and 76 of the Cr. PC also provides that a person has to be
Exp.: Article 35A is a provision incorporated in the Constitution
produced before a court within 24 hours of arrest.
giving the Jammu and Kashmir Legislature a carte
Additionally, Article 22(1) of the Constitution says an
blanche to decide who all are ‘permanent residents’ of
arrested person cannot be denied the right to consult, and
the State and confer on them special rights and privileges
to be defended by, a legal practitioner of his choice.
in public sector jobs, acquisition of property in the State,
zzBut none of these rights are available to a person scholarships and other public aid and welfare. Hence,
detained under the NSA. Hence, option (d) is correct. statement 1 is correct.
zzA person could be kept in the dark about the reasons for zzThe provision mandates that no act of the legislature
his arrest for up to five days, and in exceptional coming under it can be challenged for violating the
circumstances upto 10 days. Even when providing the Constitution or any other law of the land.
grounds for arrest, the government can withhold zzArticle 35A was incorporated into the Constitution in
information which it considers to be against public interest 1954 by an order of the then President Rajendra Prasad
to disclose. on the advice of the Jawaharlal Nehru Cabinet. The
zzThe arrested person is also not entitled to the aid of any Presidential Order was issued under Article 370 (1) (d)
legal practitioner in any matter connected with the of the Constitution. This provision allows the President
proceedings before an advisory board, which is constituted to make certain “exceptions and modifications” to the
by the government for dealing with NSA cases. Constitution for the benefit of ‘State subjects’ of Jammu
and Kashmir. Hence, statement 2 is correct.
51. (a)
zzThe controversial Constitution (Application to Jammu
Exp.: Voter Verifiable Paper Audit Trail (VVPAT) machines and Kashmir) Order of 1954 followed the 1952 Delhi
are used during election process to verify that the vote Agreement entered into between Nehru and the then
polled by a voter goes to the correct candidate. Prime Minister of Jammu and Kashmir Sheikh Abdullah,
zzVVPATs are a second line of verification and are which extended Indian citizenship to the ‘State subjects’
particularly useful in the time when allegations around of Jammu and Kashmir.
Electronic Voting Machines’ tampering crop up. zzSo Article 35A was added to the Constitution as a
zzVVPAT system gives instant feedback to the voter testimony of the special consideration the Indian
showing that the vote polled has in fact been allotted to government accorded to the ‘permanent residents’ of
the candidate chosen. Hence, statement 1 is correct. Jammu and Kashmir.

641] çFke ry] eq[kthZ uxj] fnYyh&9 13


nwjHkk"k % 011-47532596, (+91) 8448485517
bZ&esy % helpline@groupdrishti.com, osclkbV % www.drishtiias.com isQlcqd % facebook.com/drishtithevisionfoundation
Copyright – Drishti The Vision Foundation
https://telegram.me/pdf4exams https://telegram.me/ias201819

54. (c) zzThe infection is also known to affect human beings. The
Exp.: AstroSat is the first dedicated Indian astronomy mission organism which causes Nipah Virus encephalitis is an
aimed at studying celestial sources in X-ray, optical and RNA or Ribonucleic acid virus of the family
UV spectral bands simultaneously. Hence, statement 1 Paramyxoviridae, genus Henipavirus, and is closely
is correct. related to Hendra virus.
zzOne of the unique features of AstroSat mission is that it zzNipah virus infection gets its name from the village in
enables the simultaneous multi-wavelength observations Malaysia where the person from whom the virus was first
of various astronomical objects with a single satellite. isolated succumbed to the disease. Hence, statement 1
is correct.
zzThe scientific objectives of AstroSat mission are:
zzThe virus has been listed in the World Organisation for
„„To understand high energy processes in binary star
Animal Health (OIE) Terrestrial Animal Health Code and
systems containing neutron stars and black holes;
must be reported to the OIE (OIE Terrestrial Animal
„„Estimate magnetic fields of neutron stars;
Health Code).
„„Study star birth regions and high energy processes in
zzThe disease spreads through fruit bats or ‘flying foxes,’ of
star systems lying beyond our galaxy; the genus Pteropus, who are natural reservoir hosts of the
„„Detect new briefly bright X-ray sources in the sky; Nipah and Hendra viruses. Hence, statement 2 is correct.
„„Perform a limited deep field survey of the Universe zzThe virus is present in bat urine and potentially, bat faeces,
in the Ultraviolet region. Hence, statement 2 is saliva, and birthing fluids. Presumably, the first incidence
correct. of Nipah virus infection occurred when pigs in Malaysian
farms came in contact with the bats who had lost their
55. (c)
habitats due to deforestation. Furthermore, transmission
Exp.: Pangolins are mammals. They are the only mammals between farms may be due to fomites – or carrying the
wholly-covered in scales and they use those scales to virus on clothing, equipment, boots, vehicles.
protect themselves from predators in the wild.
zzThese are distributed in two continents– Asia and Africa 58. (c)
and listed as vulnerable to critically endangered on IUCN Exp.: Asoka was the first ruler who inscribed his messages to
list. his subjects and officials on stone surfaces – natural rocks
zzNandankanan Zoological Park situated in Odisha has as well as polished pillars. He used the inscriptions to
Pangolin Conservation Breeding centres, established in proclaim what he understood to be dhamma. This
2008 with financial support of India’s Central Zoo included respect towards elders, generosity towards
authority. Brahmanas and those who renounced worldly life,
treating slaves and servants kindly, and respect for
Hence, both statements are correct.
religions and traditions other than one’s own.
56. (c) Asoka also tried to hold his empire together by
Exp.: The principal International Treaty on space is the 1967 propagating dhamma. This, according to him, would
Outer Space Treaty. ensure the well-being of people in this world and the next.
zzIndia is a signatory to this treaty, and ratified it in 1982. Special officers, known as the dhamma mahamatta,
Hence, statement 2 is correct. were appointed to spread the message of dhamma.
zzThe Outer Space Treaty explacitly prohibits weapons of Hence statement 2 is correct.
mass destruction or nuclear weapons in orbit or on There were five major political centres in the empire

celestial bodies or station then in outer space or any other – the capital Pataliputra and the provincial centres of
manner. Hence, statement 1 is correct. Taxila, Ujjayini, Tosali and Suvarnagiri, all mentioned
in Asokan inscriptions. It is likely that administrative
57. (c)
control was strongest in areas around the capital and the
Exp.: Nipah Virus is an emerging infectious disease that broke provincial centres. These centres were carefully chosen,
out in Malaysia and Singapore in 1998 and 1999. both Taxila and Ujjayini being situated on important long-
zzIt first appeared in domestic pigs and has been found distance trade routes, while Suvarnagiri (literally, the
among several species of domestic animals including golden mountain) was possibly important for tapping the
dogs, cats, goats, horses and sheep. gold mines of Karnataka. Hence, statement 1 is correct.

641, First Floor, Dr. Mukherjee Nagar, Delhi-110009 14


Contacts% 011-47532596, (+91) 8448485517
E-mail: helpline@groupdrishti.com, Web: www.drishtiias.com Facebook: facebook.com/drishtithevisionfoundation
Copyright – Drishti The Vision Foundation
https://telegram.me/pdf4exams https://telegram.me/ias201819

59. (c) theory of relativity and the principles of quantum physics,


Exp.: Mechanical waves and electromagnetic waves are two Chandrasekhar showed that it is impossible for a white
important ways that energy is transported in the world dwarf star, which is supported solely by a degenerate gas
around us. Waves in water and sound waves in air are two of electrons, to be stable if its mass is greater than 1.44
examples of mechanical waves. times the mass of the Sun. If such a star does not
completely exhaust its thermonuclear fuel, then this
zzMechanical waves are caused by a disturbance or
limiting mass may be slightly larger.
vibration in matter, whether solid, gas, liquid, or
plasma. Matter that waves are traveling through is zzAll direct mass determinations of actual white dwarf stars
called a medium. Water waves are formed by vibrations have resulted in masses less than the Chandrasekhar limit.
in a liquid and sound waves are formed by vibrations in A star that ends its nuclear-burning lifetime with a mass
a gas (air). These mechanical waves travel through a greater than the Chandrasekhar limit must become either
medium by causing the molecules to bump into each other, a neutron star or a black hole.
like falling dominoes transferring energy from one to the 62. (d)
next. Sound waves cannot travel in the vacuum of space
Exp.: The term ‘Office of Profit’ has not been clearly defined
because there is no medium to transmit these mechanical
in the constitution or the Representation of People Act,
waves. Hence, statement 1 is correct.
1951. Hence, statement 1 is NOT correct.
zzElectromagnetic waves differ from mechanical waves in
that they do not require a medium to propagate. This Articles 102 (1)(a) and 191 (1)(a) of the constitution
merely state that a Member of Parliament of Member of
means that electromagnetic waves can travel not only
Legislative Assembly is barred from holding an Office
through air and solid materials, but also through the
of Profit as it can put them is a position to gain financially.
vacuum of space. Hence, statement 2 is correct.
Hence, statement 2 is NOT correct.
60. (c) In Jaya Bachan vs Union of India case (2006) the supreme
Exp.: The judicial system was far more developed under court defined an Office of profit as any office capable of
the Guptas than in earlier times. Several law books yielding a profit or pecuniary gain.
were compiled in this period. For the first time criminal
Further in the Swapan Roy case the Supreme Court laid
and civil law were clearly defined and demarcated.theft
down the following parameters to check if a post can be
and adultery came under criminal law.disputes regarding
termed an Office of Profit:
various types of property came under civil law.it was the
duty of the king to uphold the law. The king tried cases Whether the appointment has been made by the
with the help of brahamana priests.The guilds of artisans, government.
merchants and others were governed by their own laws. Whether the government has the power to dismiss or
Hence, statement 1 is correct. remove.
The most important officers in the Gupta empire were
Whether the government is paying the remuneration or
the Kumaramatyas. They were appointed by the king not.
in the home provinces and possibly paid in the cash. Since
Whether the government determines or controls the
the Guptas were possibly vaisya, recruitments were not
functions of such office
confined to upper varnas only.the Gupta bureaucracy was
not as elaborate as that of the Mauryas. Hence, statement Whether functions are performed for the government.
2 is correct. 63. (a)
61. (a) Exp.: The Members of Parliament Local Area Development
Exp.: Chandrasekhar limit, in astrophysics, maximum mass Division is entrusted with the responsibility of
theoretically possible for a stable white dwarf star. Hence, implementation of Members of Parliament Local Area
option (a) is correct. Development Scheme (MPLADS). Under the scheme,
zzThis limiting value was named for the Indian-born each MP has the choice to suggest to the District Collector
astrophysicist Subrahmanyan Chandrasekhar, who for works to the tune of Rs.5 Crores per annum to be
formulated it in 1930. Using Albert Einstein’s special taken up in his/her constituency.
641] çFke ry] eq[kthZ uxj] fnYyh&9 15
nwjHkk"k % 011-47532596, (+91) 8448485517
bZ&esy % helpline@groupdrishti.com, osclkbV % www.drishtiias.com isQlcqd % facebook.com/drishtithevisionfoundation
Copyright – Drishti The Vision Foundation
https://telegram.me/pdf4exams https://telegram.me/ias201819

The Rajya Sabha Members of Parliament can recommend The main aims of the Indian National Congress in the
works in one or more districts in the State from where initial stage were to–
he/she has been elected. (i) found a democratic, nationalist movement;
The Nominated Members of the Lok Sabha and Rajya (ii) politicise and politically educate people;
Sabha may select any one or more Districts from any one (iii) establish the headquarters for a movement;
State in the Country for implementation of their choice (iv) promote friendly relations among nationalist
of work under the scheme. political workers from different parts of the
country;
In MPLAD MP's can pick constituency (villages) which
(v) develop and propagate an anti-colonial nationalist
belong to themselves or their spouses. Hence, statement
ideology;
2 is NOT correct.
(vi) formulate and present popular demands before the
However in Sansad Adarsh Gram Yojana (SAGY) MP's government with a view to unifying the people over
cannot pick villages which belong to themselves or their a common economic and political programme;
spouses. (vii) develop and consolidate a feeling of national unity
The scheme is funded and administered through the Union among people irrespective of religion, caste or
Ministry of Statistics and Programme Implementation province.
(MoSPI). Projects are to be recommended to and (viii) carefully promote and nurture Indian nationhood.
implemented by the district-level administration. Hence, Hence, statement 2 is correct.

statement 1 is correct.
66. (b)
64. (d)
Exp.: Seva Bhoj Yojna
Exp.: Section 151A of the Representation of the People Act, zzSeva Bhoj Yojna’ is a Central Sector Scheme of the
1951 deals with time limit within which casual vacancies Ministry of Culture, Government of India. Hence,
in Parliament and State Legislatures should be filled Statement 1 is NOT correct.
through bye-elections by Election Commission of India
zzThe Scheme seeks to reimburse Central Share of CGST
(ECI). Hence, statement 2 is NOT correct.
and IGST on Food/Prasad/Langar/Bhandara offered by
The section mandates ECI to fill the vacancies within 6 Religious Institutions so as to lessen the financial burden
months from the date of occurrence of the vacancy of such Charitable Religious Institutions, who provide
provided that the remainder of the term of a member in Food/Prasad/Langar (Community Kitchen)/Bhandara free
relation to a vacancy is one year or more. Hence, of cost without any discrimination to Public/Devotees.
statement 1 is NOT correct. Hence, Statement 2 is correct.
Exception in Sec 151A - If ECI in consultation with the zzThe Charitable Religious Institutions covered under the
Central Government certifies that it is difficult to hold the scheme are Temples, Gurudwara, Mosque, Church,
bye-election within the period, then section 151A can be Dharmik Ashram, Dargah, Matth, Monasteries etc.
overruled.
67. (c)
65. (b) zzRecently World Bank released a report titled ‘South
Exp.: In the later 1870s and early 1880s, a solid ground had Asia’s Hotspot’ that analyses how climate change will
been prepared for the establishment of an all-India have a negative impact on living standards in India,
organisation. The final shape to this idea was given by a Pakistan, Bangladesh and Sri lanka.
retired English civil servant, A.O. Hume, who mobilised
Some important facts of the report:
leading intellectuals of the time and, with their cooperation,
zzThe report finds that average temperatures in the region
organised the first session of the Indian National Congress
at Gokuldas Tejpal Sanskrit College in Bombay in have increased in the last sixty years and will continue
December 1885. The first session of the Indian National rising.
Congress was attended by 72 delegates and presided over zzEight hundred million South Asians are at risk to see
by Womesh Chandra Bonnerjee. Hereafter, the their standards of living and incomes decline as rising
Congress met every year in December, in a different part temperatures and more erratic rainfalls will cut down crop
of the country each time. Hence, statement 1 is NOT yields, make water more scarce, and push more people
correct. away from their homes to seek safer places.

641, First Floor, Dr. Mukherjee Nagar, Delhi-110009 16


Contacts% 011-47532596, (+91) 8448485517
E-mail: helpline@groupdrishti.com, Web: www.drishtiias.com Facebook: facebook.com/drishtithevisionfoundation
Copyright – Drishti The Vision Foundation
https://telegram.me/pdf4exams https://telegram.me/ias201819

zzThe report projects a 2.8 per cent erosion for India’s zzAccording to the National Disaster Risk Index Maharashtra
GDP by 2050 and a fall in the living standards for nearly has been ranked at the top of the list of Indian states,
half of Indians due to change in temperature and vulnerable to natural disasters, followed by West Bengal,
precipitation pattern. Hence, option (c) is correct. Uttar Pradesh, and Madhya Pradesh. Hence, Statement
68. (a) 1 is correct.
zzAmong the Union Territories, Delhi is the most vulnerable
Exp.: Malabar Exercise
zzThe Malabar exercise was started in 1992 as a bilateral to such disasters. Hence, Statement 2 is NOT correct.
one between the Indian Navy and the US Navy in the zzNorth 24 Parganas is the most disaster prone district in
Indian Ocean. Japan became a permanent member of the India followed by Pune. Hence, Statement 3 is correct.
Malabar exercise from 2015 making it a trilateral exercise. zzThe index not only talks about the possibility of a natural
zzIt is for the first time that Malabar Exercise -2018 was disaster, but the economic vulnerability of the region and
conducted at Guam, a US territory. the steps taken by the administration to mitigate the risks.
zzThree Indian Naval Ships INS Sahyadri, INS Shakti
71. (d)
and INS Kamorta participated in the Malabar Exercise
-2018. Exp.: K2-236b
zzThis maritime exercise has grown in scope and complexity zzEPIC 211945201b or K2-236b is the first exoplanet
and aims to address the variety of shared threats and discovered by Indian researchers recently.
challenges to maritime security in the Indo-Pacific region. zzIt is six times bigger than Earth and revolving around a
zzFurther, the exercise contributes towards increasing the Sun-like star about 600 light years away.
level of mutual understanding, interoperability and zzWith this discovery India has joined a select group of
sharing of the best practices between the three navies. countries which have discovered planets around stars.
Hence, option (a) is the correct.
zzThe surface temperature of the planet is around 600°C as
69. (b) it is very close to the host star (7 times nearer than Earth-
Exp.: DISHA dashboard Sun distance).
zzIt is developed by the Ministry of Rural Development. zzThis might make it unhabitable, but such a discovery is
Hence, Statement 1 is NOT correct. of importance for understanding the formation mechanism
zzIt is a tool that will make it easier to monitor governance of such super-Neptune or sub-Saturn kind of planets, that
by geography in real time. are too close to the host star.
zzThe application is available to all members of Parliament
Hence, option (d) is correct.
and State Assemblies as well district officials, it allows
the user to track the progress of multiple and diverse 72. (b)
schemes in a certain district, block, or even a gram Exp.: Generalised System of Preferences (GSP)
panchayat. Hence, Statement 2 is correct. zzThe (GSP) is one of the oldest trade preference
zzThe dashboard has three filters: time, scheme and programmes in the world, and was designed to provide
geography. Currently, the tool is only available to zero duties or preferential access for developing countries
legislators and government officials, but the Ministry is to advanced markets.
considering the possibility of making some of its features
zzThe U.S. GSP programme was established by the U.S.
available online to the public on a later date.
Trade Act of 1974, and promotes economic development
zzThere is no such target of the initiative to promote digital
by eliminating duties on thousands of products when
transactions between different ministries. Hence,
imported from one of the 129 designated beneficiary
Statement 3 is NOT correct.
countries and territories. The GSP does not come under
70. (c) the MFN (Most favoured Nation) treatment of WTO.
Exp:. National Disaster Risk Index Hence, Statement 1 is NOT correct.
zzThe Union Ministry of Home Affairs with the support of zzFor over 40 years, GSP has fulfilled its purpose of
United Nations Development Programme (UNDP) have promoting economic growth in a large number of
prepared for the first time a national disaster risk index developing countries by allowing increased exports of
for India. eligible products. Hence, Statement 2 is correct.
641] çFke ry] eq[kthZ uxj] fnYyh&9 17
nwjHkk"k % 011-47532596, (+91) 8448485517
bZ&esy % helpline@groupdrishti.com, osclkbV % www.drishtiias.com isQlcqd % facebook.com/drishtithevisionfoundation
Copyright – Drishti The Vision Foundation
https://telegram.me/pdf4exams https://telegram.me/ias201819

zzThis tremendous benefit to the global economy is a small zzThere is no such possible benefits of decrease in siltation
aspect of the U.S. trade balance; for example, of the total of the coastal ports from the recent relaxation in cabotage
$2.4 trillion U.S. imports in 2017, only $21.2 billion law. Hence, Statement 3 is NOT correct.
arrived via GSP, amounting to less than 1% of total U.S.
imports. 74. (b)
zzGSP allows Indian exporters a certain competitive edge Financial Stability and Development Council (FSDC)
and furthers the development of the country’s export base. zzFSDC is a regulatory body for regulating financial sector
This helps Indian exporters a competitive edge for exports of the economy.
to US markets. It also allows India to integrate with global zzThe FSDC is headed by the Finance Minister and
value chains (GVC) and hence, with global markets. comprises the RBI Governor, the SEBI Chairman, and
Hence, Statement 3 is correct. heads of regulators such as PFRDA, IRDAI and the
zzMost of the 3,500 Indian products imported by the U.S. Chairman of the Insolvency and Bankruptcy Board.
under the GSP are raw materials or important intermediaries Hence, Statement 1 is NOT correct.
of value chains. In many cases, Indian exports are less-
zzThe FSDC was set up to create an institutional structure
expensive, high-quality alternatives that reduce the costs
and mechanism for financial stability, financial sector
of final products.
development and inter-regulatory coordination.
zzIt creates value chain that is subsequently exported to the
zzIt is charged with financial literacy, financial inclusion
world over by U.S. companies or directly conveyed to
the U.S. consumer. This enables the U.S. economy to be and macro prudential supervision of the economy,
more globally competitive. including the functioning of large financial conglomerates.
Hence, Statement 2 is correct.
zzOverall its benefits ultimately help U.S. consumers and
exporters by contributing to lower pricing of final 75. (d)
products.
Exp.: GDP deflator
73. (a) zzThe GDP deflator, also called implicit price deflator, is a
Exp.: Cabotage law measure of inflation. It is the ratio of the value of goods
zzIt is related to the coastal cargo movement along the and services an economy produces in a particular year at
India’s coastline. current prices to that of prices that prevailed during the
zzUnder Earlier cabotage rule, it allowed only Indian base year. Hence, Statement 1 is correct.
shipping lines to carry export-import containers for trans- zzThe deflator covers the entire range of goods and services
shipment on local routes and foreign ships could operate produced in the economy, as against the limited
along the coast only when Indian ships were not available. commodity baskets for the wholesale or consumer price
zzSo far, the cabotage policy in the country gives first indices. Therefore, it is seen as a more comprehensive
preference to Indian flagships over cargo and foreign measure of inflation. Hence, Statement 2 is correct.
vessels. zzGDP deflator is available only on a quarterly basis along
zzNow recently India relaxed its cabotage law, allowing with GDP estimates, whereas CPI and WPI data are
foreign ships to transport loaded or empty containers from released every month. Hence, Statement 3 is correct.
one domestic port to another.
76. (a)
zzThe recent policy changes are expected to increase the
domestic shipping capacity and bring down the cost of Exp.: External Commercial Borrowings
coastal transportation. Hence, Statement 1 is correct. zzRecently to improve ease of doing business, the Reserve
zzIn addition, the Shipping ministry has relaxed the coastal Bank of India has decided to liberalise External
movement of agriculture, horticulture, fisheries and Commercial Borrowing (ECB) norms, allowing all
animal husbandry commodities, a potentially significant companies that are eligible for receiving foreign direct
easing of doing business. investment, to raise funds through the ECB route. Hence,
zzSince cabotage law is not related to the coastal security Statement 1 is correct.
from the threat of terrorist attacks. Hence, Statement 2 zzThe RBI has decided to keep the minimum average
is NOT correct. maturity period at 3 years for all ECBs, irrespective of

641, First Floor, Dr. Mukherjee Nagar, Delhi-110009 18


Contacts% 011-47532596, (+91) 8448485517
E-mail: helpline@groupdrishti.com, Web: www.drishtiias.com Facebook: facebook.com/drishtithevisionfoundation
Copyright – Drishti The Vision Foundation
https://telegram.me/pdf4exams https://telegram.me/ias201819

the amount of borrowing, except for borrowers specifically 79. (c)


permitted to borrow for a shorter period. Earlier, the Exp.:
minimum average maturity period was five years. The
ceiling for borrowing remains at $750 million.
zzThe RBI has eased these norms to arrest the sharp fall in
the rupee.
zzRBI bars companies from using ECBs for working capital
needs or repaying Rupee loans. Hence, Statement 2 is
NOT correct.
77. (b)
Exp.: White Label ATMs
zzWhite Label ATMs (WLAs) are owned and operated by
non-bank entities. Hence, statement 1 is NOT correct.
zzTo run WLAs, authorisation has been given by RBI under
the Payment and Settlement Systems Act, 2007. Hence, As can be seen in the map the countries bordering the
statement 2 is correct. mediterranean sea are: Libya, Egypt, Israel, Lebonon,
zzWhite Label ATMs are aimed at increasing the geographical Syria, etc. Jordan is land locked country and does not
spread of ATMs because of inability of banking touch mediterranean sea. Hence, option (c) is correct.
institutions to provide sufficient number of ATMs 80. (b)
facilities. Hence, statement 3 is correct.
Exp.: In India, the mangrove forests spread over 6,740 sq. km
zzIt is not permissiable for white label ATMs to take over
which is 7 per cent of the world’s mangrove forests. They
an existing ATM of any other bank.
are highly developed in the Andaman and Nicobar Islands
zzWhite Label ATM operators are responsible for the
and the Sunderbans of West Bengal.
maintenance and service regarding facilities of WLAs.
zzThe deltas of the Ganga, the Mahanadi, the Krishna, the
78. (c) Godavari and the Kaveri are covered by such vegetation.
Exp.: The Indus is one of the largest river basins of the world, In the Ganga-Brahmaputra delta, sundari trees are found.
covering an area of 11,65,000 sq. km (in India it is 321, Hence, statement 1 is NOT correct.
289 sq. km) and a total length of 2,880 km (in India zzThe mangrove tidal forests are found in the areas of coasts
1,114 km). It originates from a glacier near Bokhar Chu influenced by tides. Mud and silt get accumutated on such
(31°15' N latitude and 81°40' E longitude) in the Tibetan coasts. Dense mangroves are the common varieties with
region at an altitude of 4,164 m in the Kailash Mountain roots of the plants submerged under water
range. In Tibet, it is known as ‘Singi Khamban; or Lion’s zzMangroves grow along the coasts in the salt marshes,
mouth. After flowing in the northwest direction between tidal creeks, mud flats and estuaries. They consist of a
the Ladakh and Zaskar ranges, it passes through number of salt-tolerant species of plants. Crisscrossed by
Ladakh and Baltistan. Hence , statement 1 is NOT creeks of stagnant water and tidal flows, these forests give
correct. shelter to a wide variety of birds. Hence, statement 2 is
zzThe Satluj originates in the ‘Raksas tal’ near Mansarovar correct.
at an altitude of 4,555 m in Tibet where it is known as
Langchen Khambab. It flows almost parallel to the Indus 81. (c)
for about 400 km before entering India, and comes out Exp.: The Himalayan ranges show a succession of vegetation
of a gorge at Rupar. from the tropical to the tundra, which change in with the
zzThe Satluj passes through the Shipki La on the Himalayan altitude.
ranges and enters the Punjab plains. It is an antecedent zzIn the higher hill ranges of northeastern India, hilly areas
river. Hence, statement 2 is NOT correct while of West Bengal and Uttarakhand, evergreen broad leaf
statement 3 is correct. trees such as oak and chestnut are predominant.
641] çFke ry] eq[kthZ uxj] fnYyh&9 19
nwjHkk"k % 011-47532596, (+91) 8448485517
bZ&esy % helpline@groupdrishti.com, osclkbV % www.drishtiias.com isQlcqd % facebook.com/drishtithevisionfoundation
Copyright – Drishti The Vision Foundation
https://telegram.me/pdf4exams https://telegram.me/ias201819

zzBetween 1,500-1,750 m, pine forests are also well- zzIntroduction of Doctrine of Lapse by Lord Dalhousie was
developed in this zone, with Chir Pine as a very useful the political reason behind the revolt. Heavy taxation,
commercial tree. Deodar, a highly valued endemic species discriminatory tariff policy against Indian products,
grows mainly in the western part of the Himalayan range. destruction of traditional handicrafts and forcibly
zzThe chinar and the walnut, which sustain the famous evictions were the underlying economic causes. Hence,
Kashmir handicrafts, belong to this zone. Blue pine and statement 1 and 2 are correct.
spruce appear at altitudes of 2,225-3,048 m. zzReforms introduced by the British East India Company
zzSilver firs, junipers, pines, birch and rhododendrons, in the social and religious dispensation harmed the
etc. occur between 3,000-4,000 m. in Montane Forests sentiments of orthodox and conservative people, and
The southern slopes of the Himalayas carry a thicker hence contributed in the emergence of the revolt. Hence,
vegetation cover because of relatively higher precipitation statement 3 is correct.
than the drier north-facing slopes. At higher altitudes, zzThe introduction of Enfield rifles whose cartridges were
mosses and lichens form part of the tundra vegetation. said to have a greased cover made of beef and pork sparked
zzThe Moist deciduous forests are found in the northeastern off the revolt, and is considered the immediate cause.
states along the foothills of Himalayas, eastern slopes of
84. (a)
the Western Ghats and Odisha. Teak, sal, shisham, hurra,
mahua, amla, semul, kusum, and sandalwood etc. are the Exp.: During 1917 and 1918, Gandhi was involved in three
main species of these forests. Hence , option (c) is correct. struggles – in Champaran, Ahmedabad and Kheda.
zzGandhi was requested by Rajkumar Shukla, a local man,
82. (b)
to look into the problems of the farmers in context of
Exp: Dual Government of Bengal (1765–1772) indigo planters of Champaran in Bihar.
zzIt was introduced by Robert Clive in Bengal in 1765. The
zzThe European planters had been forcing the peasants to
new arrangements provided the British East India
grow indigo on 3/20 part of the total land (called tinkathia
Company with Diwani (Fiscal) rights, and Nizamat
system).
(territorial) rights were entrusted to the Nawabs. Hence,
zzWhen Gandhi reached Champaran to probe into the
statement 1 is NOT incorrect.
matter, the authorities ordered him to leave the area at
zzRobert Clive appointed Mohammad Raza Khan as the
once. Gandhi defied the order and preferred to face the
Diwan of Bengal and Raja Shitab Roy as the Diwan of
punishment.
Bihar. Hence, statement 2 is NOT incorrect.
zzThis passive resistance or civil disobedience of an unjust
zzThe system of dual government was ended by Warren
order was a novel method at that time. So champaran
Hastings in 1772. Hence, statement 3 is correct.
satyagraha movement was first civil disobedience.
zzUnder this system, Indian merchants lost grounds as they
Hence , pair 1 is correctly matched.
had to pay taxes but in case of British merchants it became
zzIn March 1918, Gandhi intervened in a dispute between
easy as they were provided duty-free trade opportunities.
cotton mill owners of Ahmedabad and the workers over
zzPeasant class also suffered tremendously because of the
the issue of discontinuation of the plague bonus.
high demand of revenue by the British East India Company.
zzGandhi advised the workers to remain non-violent while
83. (d) on strike. When negotiations with mill owners did not
Exp.: 1857 Revolt progress. He himself undertook a fast unto death (his first)
zzIt is considered to be the first ever struggle against the to strengthen the workers’ resolve. But the fast also had
inhuman rule of the British East India Company. Political, the effect of putting pressure on the mill owners who
Social and Socio-religious causes impacted the sentiments finally agreed to submit the issue to a tribunal. It was first
of the people of India which culminated in unleashing hunger strike by Gandhi. Hence, pair 2 is NOT
strong resistance against British government. correctly matched.

641, First Floor, Dr. Mukherjee Nagar, Delhi-110009 20


Contacts% 011-47532596, (+91) 8448485517
E-mail: helpline@groupdrishti.com, Web: www.drishtiias.com Facebook: facebook.com/drishtithevisionfoundation
Copyright – Drishti The Vision Foundation
https://telegram.me/pdf4exams https://telegram.me/ias201819

zzBecause of drought in 1918, the crops failed in Kheda zzTo express people’s feeling that they were not interested
district of Gujarat. According to the Revenue Code, if the in the war and that they made no distinction between
yield was less than one-fourth the normal produce, the Nazism and the double autocracy that ruled India.
farmers were entitled to remission. zzTo give another opportunity to the government to accept
zzGandhi asked the farmers not to pay the taxes. Gandhi, Congress’demands peacefully.
however, was mainly the spiritual head of the struggle. zzVinoba Bhave was the first to offer the satyagraha
zzUltimately, the government sought to bring about an followed by Pandit Jawahar lal Nehru. Hence, Statement
agreement with the farmers. It agreed to suspend the tax 1 is correct.
for the year in question, and for the next; reduce the zzThe demand of the satyagrahi would be the freedom of
increase in rate; and return all the confiscated property. speech against the war through an anti-war declaration.
It was the first Non-cooperation movement. Hence, zzIf the government did not arrest the satyagrahi, he or she
pair 3 is NOT correctly matched. would not only repeat it but move into villages and start
85. (c) a march towards Delhi, thus precipitating a movement
which came to be known as the ‘Delhi Chalo Movement’.
Exp.: Rawlatt Act
Hence, Statement 2 is correct.
zzThe act allowed political activists to be tried without juries
or even imprisoned without trial. It allowed arrest of 87. (d)
Indians without warrant on the mere suspicion of Exp.: Quit India Resolution
‘treason’. zzIn July 1942, the Congress Working Committee met at
zzSuch suspects could be tried in secrecy without recourse Wardha and a resolution was passed that demanded that
to legal help. the British Rule in India must end immediately.
zzA special cell consisting of three high court judges was zzIt authorised Gandhi to take the charge of the non-violent
to try such suspects and there was no court of appeal mass movement.
above that panel. This panel could even accept evidence zzThis resolution is generally referred to as the ‘Quit India’
not acceptable under the Indian Evidences Act. resolution and was proposed by Jawaharlal Nehru. Hence,
zzThe Law of Habeas Corpus, the basis of civil liberty, was Statement 1 is correct.
sought to be suspended. Hence , statement 1 is correct. zzThe Quit India Resolution was ratified at the Congress
zzGandhi called for a mass protest at all India level. But meeting at Gowalia Tank, Bombay, on August 8, 1942.
soon, having seen the constitutional protest meet with zzThis resolution taken at Wardha was known as Wardha
ruthless repression. Resolution and is also known as “Quit India Resolution”.
zzGandhi organised a Satyagraha Sabha and roped in  younger Hence, Statement 2 is correct.
members of Home Rule Leagues and the Pan Islamists.
88. (c)
zzThe forms of protest finally chosen included observance
Exp.: Cabinet Mission Plan
of a nationwide strike accompanied by fasting and prayer,
zzThe mission arrived in India on March 24, 1946.
and civil disobedience against specific laws, and courting
arrest and imprisonment. Hence , statement 2 is correct. zzIts objective was to devise a machinery to draw up the
constitution of Independent India and make arrangements
86. (c) for interim Government.
Exp.: Towards the end of 1940, Gandhi decided to initiate a
Its main points are
limited satyagraha on an individual basis by a few selected zzRejection of the demand for a full-fledged Pakistan
individuals in every locality. because the Pakistan so formed would include a large
The aims of launching individual satyagraha were– non-Muslim population in the North-West and in the
zzTo show that nationalist patience was not due to weakness. North-East. Hence, Statement 1 is NOT correct.

641] çFke ry] eq[kthZ uxj] fnYyh&9 21


nwjHkk"k % 011-47532596, (+91) 8448485517
bZ&esy % helpline@groupdrishti.com, osclkbV % www.drishtiias.com isQlcqd % facebook.com/drishtithevisionfoundation
Copyright – Drishti The Vision Foundation
https://telegram.me/pdf4exams https://telegram.me/ias201819

zzGrouping of existing provincial assemblies into three 91. (c)


sections of provinces. Exp.: Communal Award
zzA common centre would control defence, communication zzThe Communal Award was announced by the British
and external affairs. A federal structure was envisaged for prime minister, Ramsay MacDonald, on August 16, 1932.
India.
zzIt is based on the findings of the Indian Franchise
zzCommunal questions in the central legislature were to be Committee (also called the Lothian Committee).
decided by a simple majority of both communities present
zzIt accorded separate electorates for Muslims, Europeans,
and voting.
Sikhs, Indian Christians, Anglo-Indians, depressed classes,
zzProvinces were to have full autonomy and residual and even to the Marathas for some seats in Bombay.
powers. Hence, Statement 2 is correct.
zzJain’s were not accorded seperate electorates under the
zzThree-tier executive and legislature at provincial, section Communal Award. Hence, option (c) is correct.
and union levels. Hence, Statement 3 is correct.
92. (a)
89. (d)
Exp.: Hindustan Republican Association/Army(HRA)– The
Exp.: Important Ministers of Interim Government
HRA was founded in October 1924 in Kanpur by
(September 2, 1946–August 15, 1947)
Ramprasad Bismil, Jogesh Chandra Chatterjee and Sachin
1. Jawaharlal Nehru: Vice President of Executive
Sanyal with an aim to organise an armed revolution to
Council, External Affairs and Common Wealth
overthrow the colonial government.
Relations
zzMurder of Saunders by Revolutionaries-(Dec-1928)–
2. Vallabhbhai Patel: Home, Information and
The death of Sher-i-Punjab Lala Lajpat Rai due to lathi
Broadcasting
blows received during a lathi- charge on an anti-Simon
3. Baldev Singh: Defence
Commission procession (October 1928) led revolutionaries
4. Dr. John Mathai: Industries and Supplies
once again to take to individual assassination. Bhagat
5. C. Rajagopalachari: Education
Singh, Azad and Rajguru shot dead Saunders, the police
6. C.H. Bhabha: Works, Mines and Power
official responsible for the lathicharge in Lahore in Dec-
7. Rajendra Prasad: Agriculture and Food
1928.
8. Jagjivan Ram: Labour
Hence pair 1 and 2 are not correctly matched. zzChittagong Armoury Raid (April 1930)– The raid was
Pair 3 is correctly matched. conducted in April 1930 and involved 65 activists under
the banner of Indian Republican Army – Chittagong
90. (d)
Branch by Surya Sen and his associates.
Exp.: In February 1937, elections to the provincial assemblies zzBhagat Singh, Sukhdev and Rajguru were tried in the
were held. Elections were held in eleven provinces— Lahore conspiracy case and were hanged on March 23,
Madras, Central Provinces, Bihar, Orissa, United
1931. Hence, option (a) is the correct.
Provinces, Bombay Presidency, Assam, NWFP, Bengal,
Punjab and Sindh. 93. (d)
zzThe Congress won 716 out of 1,161 seats it contested. Exp.: Indian Coast Guard was constituted in 1977 with
zzExcept in Bengal, Assam, Punjab, Sindh and the following charter of duties:
NWFP, it got a majority in all provinces. Hence, zzSafety and protection of artificial islands, offshore
Statement 1 is NOT correct. terminals and installations;
zzThe Congress emerged as the largest party in Bengal, zzProtection to fishermen including assistance to them at
Assam and the NWFP and due to its spectular performance sea while in distress;
its prestige rose considerably. Hence, Statement 2 is zzPreserve and protect maritime environment and to prevent
NOT correct. and control marine pollution;
641, First Floor, Dr. Mukherjee Nagar, Delhi-110009 22
Contacts% 011-47532596, (+91) 8448485517
E-mail: helpline@groupdrishti.com, Web: www.drishtiias.com Facebook: facebook.com/drishtithevisionfoundation
Copyright – Drishti The Vision Foundation
https://telegram.me/pdf4exams https://telegram.me/ias201819

zzAssisting Customs and other authorities in anti-smuggling a comprehensive instrument for school evaluation leading
operations; to school improvement. Hence statement 1 is correct.
zzSafety of life and property at sea and collection of’ zzShaala Darpan: The “Shaala Darpan Project” to cover
scientific data; enforcing the provisions of various all the 1099 Kendriya Vidyalayas was launched in June,
enactments in the maritime zones. Hence, option (d) is 2015. The objective of this project is to provide services
correct answer. based on school management systems to students, parents
and communities. Hence statement 2 is correct.
94. (d)
zzGIS Mapping: To ensure universal access to schools
Exp: The Sarva Shiksha Abhiyan (SSA) Programme is being including secondary schools within a reasonable distance
implemented since 2001 for universalization of of any habitation and without any discrimination, the
elementary education. It has made significant progress in geographic coordinates of school along with the school
achieving near universal access and equity. information available in UDISE is being uploaded on the
school GIS Web enabled platform i.e. http://schoolgis.nic.
Sub-Programmes under SSA
in. All states have conducted GIS mapping and shared
zzThe Padhe Bharat Badhe Bharat (PBBB), a sub-
geographical coordinates of schools with the NIC except
programme of the SSA, in classes I and II is focusing on Jammu and Kashmir.
foundational learning in early grades with an emphasis
on reading, writing and comprehension and mathematics. 96. (a)
zzThe Rashtriya Aavishkar Abhiyan (RAA), also under Exp: To improve adult education, the National Literacy
the SSA, aims to motivate and engage children of the age Mission (NLM) was launched in 1988 for the adults in
group 6-18 years, in science, mathematics and technology the age group of 15+. The programme of National Literacy
Mission was re-vamped in 2009 in alignment with new
by observation, experimentation, inference drawing and
paradigms of lifelong learning as Saakshar Bharat.
model building, through both inside and outside
zzThe National Literacy Mission Authority (NLMA) is
classroom activities.
the operating and implementing organization at national
zzVidyanjali, another sub-programme under SSA, was level for all the activities envisaged in Adult Education
launched to enhance community and private sector and Skill Development. During the 12th Five Year Plan,
involvement in Government run elementary schools NLMA has been striving to raise the literacy rate to 80  per
across the country. cent and to reduce the gender gap to less than 10 per cent.
zzShaGun portal – an Initiative to monitor the Hence, option (a) is correct.
implementation of SSA: MHRD has developed a web
97. (d)
portal called ShaGun (from the words Shaala and
Gunvatta). Exp: Copyright Board is a quasi-judicial body under the
Copyright Act, 1957. It was constituted in September,
95. (c) 1958 and had been functioning on a part-time basis.
Exp: Rashtriya Madhyamik Shiksha Abhiyan (RMSA) was Hence, statement 1 is NOT correct.
zzThe jurisdiction of the Copyright Board extends to the
launched in 2009 with the objective to enhance access to
whole of India. Hence, statement 2 is NOT correct.
secondary education and improve its quality. The schemes
zzThe Board is entrusted with the task of adjudication of
envisages to enhance the enrolment at secondary stage disputes pertaining to copyright registration and
by providing a secondary school with a reasonable assignment of copyright, rectification of registration,
distance of habitation, with in an aim to ensure Gross grant of compulsory licences in respect of works withheld
Enrollment Ratio (GER) of 100 per cent and universal from public, unpublished Indian works, for benefit of
retention by 2020. physically disabled persons, production and publication
of translations and works for certain specified purposes.
Some of the significant initiatives, under RMSA for
zzThe Copyright (Amendment) Act, 2012 provides for a
improving quality of education are as:- three member permanent Copyright Board consisting of
zzShaala Siddhi: School Standards and Evaluation a Chairman and two other members.
Framework and its web portal was launched in 2015. It is

641] çFke ry] eq[kthZ uxj] fnYyh&9 23


nwjHkk"k % 011-47532596, (+91) 8448485517
bZ&esy % helpline@groupdrishti.com, osclkbV % www.drishtiias.com isQlcqd % facebook.com/drishtithevisionfoundation
Copyright – Drishti The Vision Foundation
https://telegram.me/pdf4exams https://telegram.me/ias201819

98. (a) 100. (a)


Exp: Govt. of India has launched Deendayal Upadhyaya Gram Exp.: The Morley-Minto (or Minto-Morley) Reforms that
Jyoti Yojana (DDUGJY) for the rural areas with the translated into the Indian Councils Act of 1909 introduce
following components:
for the first time, separate electorates for Muslims for
zzSeparation of agriculture and non-agriculture feeders
facilitating judicious rostering of supply to agricultural election to the central council was established— a most
& non-agricultural consumers in the rural areas. Hence, detrimental step for India.
statement 1 is correct.
Besides separate electorates for the Muslims,
zzStrengthening and augmentation of sub-transmission &
representation in excess of the strength of their population
distribution (ST&D) infrastructure in rural areas. Hence,
statement 2 is NOT correct. was accorded to the Muslims. Also, the income
zzmetering at distribution transformers, feeders and qualification for Muslim voters was kept lower than that
consumers end. for Hindus. Hence, statement 1 is correct.
99. (c) The number of elected members in the Imperial
Exp: Total installed capacity of power stations in India stood Legislative Council and the Provincial Legislative
at 350.16 Gigawatt (GW) as of February 2019. Councils was increased.The elected members were to be
zzIndia is world's third largest producer and fourth largest indirectly elected.
consumer of electricity in the world. Hence, statement
1 is correct. The local bodies were to elect an electoral college, which
zzIndia’s rank jumped to 24 in 2018 from 137 in 2014 on in turn would elect members of provincial legislatures,
World Bank’s Ease of doing business - “Getting who in turn would elect members of the central legislature.
Electricity” ranking. Hence, statement 2 is correct. Hence, statement 2 is NOT correct.

641, First Floor, Dr. Mukherjee Nagar, Delhi-110009 24


Contacts% 011-47532596, (+91) 8448485517
E-mail: helpline@groupdrishti.com, Web: www.drishtiias.com Facebook: facebook.com/drishtithevisionfoundation
Copyright – Drishti The Vision Foundation

You might also like